Q20

 
zuckermaus22
Thanks Received: 0
Vinny Gambini
Vinny Gambini
 
Posts: 3
Joined: December 28th, 2009
 
 
 

Q20

by zuckermaus22 Tue Jan 26, 2010 12:47 pm

That diagram made it a lot easier, thank you. I am still having trouble with #20, though. In order to make it work, I violate either the "KL" rule or the M > O1.
 
aileenann
Thanks Received: 227
Atticus Finch
Atticus Finch
 
Posts: 300
Joined: March 10th, 2009
 
 
 

Re: PT 37, S3, G4 - Swim Team Game

by aileenann Tue Jan 26, 2010 4:01 pm

Let's take a look.

If J is in 8 then we know that he is also in 3. Since we need at least one OJ, the only way for that to now happen is that O is in 2 and 7. Then, since we know O is in 2 and that M's first lap is before O's first lap, we also know that M must go in 1 and also in 6. Then we have only two unassigned swimmers. However, because of their relationship to one another (no KL) we know the ordering of these two, if they have to be in consecutive spots, as in this situation, will have to be LK. So now we actually know the entire order. So the answer is (A) - for all ten laps, we knw which swimmer is assigned to the lap.

Does that make sense? Again, it's easiest to do this by realizing that mostly we only have to focus on the first 5, and only look at the rest to the extent that the 5/6 consecutive slots make things a little bit more complicated than an ordinary numbering game.

I hope this helps. Again, feel free to follow up if you have more questions on this game.